-
Notifications
You must be signed in to change notification settings - Fork 0
/
Aufgabe6.tex
80 lines (71 loc) · 3.4 KB
/
Aufgabe6.tex
1
2
3
4
5
6
7
8
9
10
11
12
13
14
15
16
17
18
19
20
21
22
23
24
25
26
27
28
29
30
31
32
33
34
35
36
37
38
39
40
41
42
43
44
45
46
47
48
49
50
51
52
53
54
55
56
57
58
59
60
61
62
63
64
65
66
67
68
69
70
71
72
73
74
75
76
77
78
79
80
\documentclass[crop=false]{standalone}
\usepackage[utf8]{inputenc}
\usepackage{amsmath}
\usepackage[dvipsnames]{xcolor}
\usepackage{pdfpages}
\usepackage{enumerate}
\usepackage{amssymb}
\usepackage[framemethod=default]{mdframed}
\usepackage[nomarginpar,left=2cm,right=2cm,top = 2cm, bottom = 2cm]{geometry}
\renewcommand{\thesubsection}{\thesection.\alph{subsection}}
\renewcommand{\thesubsubsection}{\thesection.\alph{subsection}.\roman{subsubsection}}
\mdfdefinestyle{theoremstyle}{%
linecolor=black,linewidth=.3pt,%
frametitlerule=true,%
frametitlebackgroundcolor=blue!5,
innertopmargin=\topskip,nobreak=true,
}
\mdfdefinestyle{style2}{frametitle={},%
linewidth=.3pt,topline=true,backgroundcolor=blue!3!green!8!}
\mdtheorem[style=theoremstyle]{task}{Angabe}
\newmdenv[style = style2,title=false]{solution}
\begin{document}
\begin{task}
Gegeben ist das LTI-System
\[
\begin{aligned} \dot{\mathbf{x}} &=\left[\begin{array}{cc}{-1} & {-4} \\ {1} & {3}\end{array}\right] \mathbf{x}+\left[\begin{array}{c}{3} \\ {-1}\end{array}\right] u \\ y &=\left[\begin{array}{cc}{0} & {2}\end{array}\right] \mathbf{x}-3 u \end{aligned}
\]
\begin{enumerate}[i]
\item Zeigen Sie, dass das System vollständig beobachtbar ist, und entwerfen Sie einen
vollständigen Beobachter so, dass die Eigenwerte des Fehlersystems bei $\{-2,-3\}$
liegen.
\begin{solution}
Prüfen ob das System vollständig beobachtbar ist, ob $\mathcal{O}$ den ganzen $\mathbb{R}^2$ aufspannt:
\[\mathcal{O} = \text{span}\left\{
\begin{pmatrix}0 & 2\end{pmatrix},\begin{pmatrix}2 & 6\end{pmatrix}\right\} = \mathbb{R}^2 \]
Berechnen der Koeffizienten des gewünschten charakteristischen Polynoms:
\[p(s) = \left( s+3 \right) \left( s+2 \right) = s^2 + \alpha_1 s + \alpha_0 =s^2 + 5s + 6 \rightarrow \alpha_0 = 6, \ \alpha_1 = 5 \]
Berechnen des Vektors $\hat{\mathbf{t}}_1$:
\[
\mathbf{e}_2 = \mathbf{M}_O \hat{\mathbf{t}}_1 =
\begin{pmatrix}0 \\ 1\\\end{pmatrix} =
\begin{pmatrix}0 & 2\\ 2 & 6\end{pmatrix}\hat{\mathbf{t}}_1
\rightarrow \hat{\mathbf{t}}_1 = \begin{pmatrix}\frac{1}{2} \\ 0\end{pmatrix}
\]
Berechnung von $\hat{\mathbf{k}}$:
\[\hat{\mathbf{k}} = - \alpha_0 \hat{\mathbf{t}}_1 - \alpha_1 \mathbf{A} \hat{\mathbf{t}}_1 - \mathbf{A}^2 \hat{\mathbf{t}}_1 = \begin{pmatrix} 1 \\ -\frac{7}{2}\end{pmatrix}\]
Damit kann der vollständige Beobachter berechnet werden:
\[
\begin{aligned} \dot{\mathbf{z}} &=\left(\mathbf{A}+\hat{\mathbf{k}} \mathbf{c}^{T}\right) \mathbf{z}+(\mathbf{b}+\hat{\mathbf{k}} d) u-\hat{\mathbf{k}} y \\ \hat{\mathbf{x}} &=\mathbf{E} \mathbf{z} \end{aligned} \quad \rightarrow
\begin{aligned} \dot{\mathbf{z}} &=
\begin{pmatrix}
-1 & -2 \\ 1 & -4
\end{pmatrix} \mathbf{z}+
\begin{pmatrix}
0 \\ \frac{19}{2}
\end{pmatrix}
u-
\begin{pmatrix}
1 \\ -\frac{7}{2}
\end{pmatrix}
y \\ \hat{\mathbf{x}} &=\mathbf{E} \mathbf{z} \end{aligned}
\]
\end{solution}
\item Welche Gröben des Systems müssen Sie kennen, um den Beobachter aus Punkt
(i) einsetzen zu können? Begründung!
\begin{solution}
Der Eingang $u$ und der Ausgang $y$ der Strecke sind Eingänge des Beobachters, müssen also bekannt sein. Zur Berechnung des Beobachters müssen $\mathbf{A}, \mathbf{b}, \mathbf{c}^T$ und $d$ bekannt sein. Also muss jede Größe des Beobachters bekannt sein, außer die Zustände $\mathbf{x}$ der Strecke.
\end{solution}
\end{enumerate}
\end{task}
\end{document}